Formule $\sum(\zeta(s)-1)=1$ ?

2»

Réponses

  • Avec tout mon respect, je ne comprends pas ce que vous cherchez, mais comme un résumé, je peux dire ça.
    Parfois la somme d'une infinité des rationnels donne un nombre rationnel, comme le cas des séries géométriques.
    Parfois la somme d'une infinité des rationnels donne un nombre irrationnel, comme lorsque on se sert de la formule de Taylor pour calculer le nombre pi, ou bien e.
    Et parfois la somme d'une infinité des irrationnels donne un nombre rationnel, comme pour les zeta de (2s)-1 qui donne 3/4.
    Donc tout est possible, mais tirer un nombre à partir d'une telle somme ! ça ne conduit à rien, c'est même banal pour moi.
    Cordialement.

    [Toute phrase commence par une majuscule. ;-) AD]
  • Salut.
    Sachant que.
    $$\sum_{s=2}^\infty[\zeta(s)-1]=1$$
    Par definition de Zéta $\zeta$.
    $$\sum_{s=2}^\infty[[\sum_{k=1}^\infty\frac{1}{k^s}]-1]=1$$
    Que l'on peu réécrire grâce au produit Eulérien.
    PS: $P_{(l)}$ est la suite des nombres premiers.
    $$\sum_{s=2}^\infty[[\prod_{l=1}^\infty1+\frac{1}{P_{(l)}^s}]-1]=1$$
    En développant on obtiens un truc monstrueux que l'on peu réécrire.
    $$\sum_{s=2}^\infty\sum_{l=1}^\infty\sum_{r=1}^\infty\sum_{m=0}^\infty\frac{1}{\prod_{l=1}^r P_{(l+m)}^s}=1$$
    Si je ne me trompe.
    Et du coup on peu dire en simplifiant un peu que.
    $$\zeta(s)=\sum_{k=1}^\infty\frac{1}{k^s}=1+\sum_{l=1}^\infty\sum_{r=1}^\infty\sum_{m=0}^\infty\frac{1}{\prod_{l=1}^r P_{(l+m)}^s}$$
    Et la ça ressemble davantage au crible d’Ératosthène.
    J’espère ne pas avoir fait d’erreur.
    Je relirais demain et méditerais quel sens cela peut il avoir s'il y en a un.
    Ha mince se n'est pas bon il y a erreur il faut que je trouve une écriture plus adapté.
    Quelque chose me fait penser qu'il faut aller chercher du coté des "nombres de Stirling de première espèce non signés"
    Mais pour le moment j'y comprend rien.
  • Fly7
    Modifié (November 2021)
    $\prod_{l=1}^\infty 1+\frac{1}{P_{(l)}^s}=1+1/2+1/3+1/3.2+1/5+1/5.2+1/5.3+1/5.3.2+1/7+1/7.2+1/7.3+1/7.3.2+1/7.5+1/7.5.2+1/7.5.3+1/7.5.3.2+...$
    $$\prod_{l=1}^\infty 1+\frac{1}{P_{(l)}^s}=1+1/2+1/3+1/5+1/6+1/7+1/10+1/11+1/13+1/14+1/15+...$$
    


    Salut.

    Si je développe puis je range dans l'ordre croisant, il me manque 1/4, 1/8, 1/9, 1/12, 1/16, 1/18, 1/20, etc...

    Tous les diviseurs qui ont des puissances de nombres premiers, c'est assez normal mais ça me surprend!

    😲

    Pas vous?

    Mhhhhhhhhhh

    J'ai compris mon erreur.

    $$\zeta(s)=\sum_{k=1}^\infty\frac{1}{k^s}=\prod_{l=1}^\infty\frac{P_{(l)}^s}{P_{(l)}^s-1}$$
    non pas $$\prod_{l=1}^\infty1+\frac{1}{P_{(l)}^s}=\prod_{l=1}^\infty\frac{P_{(l)}^s+1}{P_{(l)}^s}$$

    Peut être que dans mon erreur il serait intéressant de connaitre la partie manquante 1/4, 1/8, 1/9, 1/12, etc...

    Pas évident pour quelqu’un comme moi aussi nul en Mathématique.

  • Fly7
    Modifié (December 2021)

    Salut.
    Je pense avoir trouvé une formule mieux écrite mais je doute au niveau des puissances s.
    P(x) est la suite des nombres premiers commençant par 1.
    Exemple : (1, 2, 3, 5, 7, 11, 13, 17, ...)

    Ne me tapez pas sur les doigts ça permet à l'algorithme de fonctionner.
    Dommage que je n'arrive pas à rendre le latex visuel.
    Laissez-moi un peu de temps pour méditer sur cette histoire de puissance.

    PS : les banderoles du site en rouges vous ne les sentez pas un peu agressives ?

    désolé d'avoir éditer 1000 fois pas évident le truc.

    Il ne me reste plus qu'a voir cette histoire de puissance.

    haaaaaaaaaaaaaaaaaaaaaaaaa non ça fonctionne pas encore.

    J'avais un bon angle d'attaque mais y'a toujours un truc qui fonctionne pas.

    On verra demain.

    C'est sacrément bien emmêler se truc

    Se que je cherche reviens a connaitre la Décomposition en produit de facteurs premiers.
    Chose qui n'est pas si simple.

    $$\sum_{m=1}^\infty\frac{1}{m^s}\gt1+\sum_{l=0}^\infty\sum_{k=1}^\infty\frac{1}{P_{(k)}^l}(-1+\prod_{n=0}^\infty 1+\frac{1}{P_{(n+k)}^s})$$

    $$\sum_{m=1}^\infty\frac{1}{m^s}=1+\sum_{l=0}^\infty\sum_{k=1}^\infty\frac{1}{P_{(k)}^l}(-1+\prod_{n=0}^\infty 1+\frac{1}{P_{(n+k)}^s})$$
  • Fly7
    Modifié (December 2021)

    Pour la énième fois, si je ne me suis pas trompé.

    $$\sum_{k=1}^{\infty}\frac{1}{k^s}>1+(\sum_{a=1}^{\infty}\frac{1}{P_{(a)}^s})+\sum_{b=1}^{\infty}\sum_{c=2}^{\infty}\frac{1}{(P_{(b)}^c)^s}+\sum_{d=1}^{c-1}\frac{1}{(P_{(b)}^{c-d})^s}\times\sum_{e=1}^{b-1}\frac{1}{(P_{(e)}^d)^s}$$

    La formule a une lacune, il maque la famille des produits qui ont plus de deux premiers.

    Exemple:

    $\frac{1}{2^2\times3^5\times5^4}$

    Report du case tète a un autre jour.

  • Sauf erreur on a aussi,126050
    Pour la première identité https://math.stackexchange.com/questions/1929731/does-sum-n-2-infty-zetan-1-converge?noredirect=1  voir la réponse de Jack D’Aurizio 


  • D’autres identités  https://en.wikipedia.org/wiki/Riemann_zeta_function#Infinite_series dans le paragraphe Applications
  • Fly7
    Modifié (December 2021)

    Cette fois-ci c'est la bonne.

    Sans multiple : $$\sum_{k=1}^{\infty}\frac{1}{k^s}>1$$

    Pour  un seul multiple : $$\sum_{k=1}^{\infty}\frac{1}{k^s}>\sum_{a=1}^{\infty}\frac{1}{P_{(a)}^s}$$

    Deux : $$\sum_{k=1}^{\infty}\frac{1}{k^s}>(\sum_{c=1}^{\infty}\frac{1}{(P_{(c)}^2)^s})+\sum_{b=1}^{\infty}\frac{1}{P_{(b+1)}^s}\times\sum_{a=1}^{b}\frac{1}{P_{(a)}^s}$$

    Trois : $$\sum_{k=1}^{\infty}\frac{1}{k^s}>(\sum_{e=1}^{\infty}\frac{1}{(P_{(e)}^3)^s})+\sum_{d=1}^{\infty}\frac{1}{P_{(d+2)}^s}\times((\sum_{c=1}^{d}\frac{1}{(P_{(c)}^2)^s})+\sum_{b=1}^{d}\frac{1}{P_{(b+1)}^s}\times\sum_{a=1}^{b}\frac{1}{P_{(a)}^s})$$

    Une infinité de multiples : $$\sum_{k=1}^{\infty}\frac{1}{k^s}>...\times((\sum_{e=1}^{g}\frac{1}{(P_{(e)}^3)^s})+\sum_{d=1}^{g}\frac{1}{P_{(d+2)}^s}\times((\sum_{c=1}^{d}\frac{1}{(P_{(c)}^2)^s})+\sum_{b=1}^{d}\frac{1}{P_{(b+1)}^s}\times\sum_{a=1}^{b}\frac{1}{P_{(a)}^s}))$$

    Puis sommer le tout :

    $$\sum_{k=1}^{\infty}\frac{1}{k^s}=1+(\sum_{a=1}^{\infty}\frac{1}{P_{(a)}^s})+((\sum_{c=1}^{\infty}\frac{1}{(P_{(c)}^2)^s})+\sum_{b=1}^{\infty}\frac{1}{P_{(b+1)}^s}\times\sum_{a=1}^{b}\frac{1}{P_{(a)}^s})+((\sum_{e=1}^{\infty}\frac{1}{(P_{(e)}^3)^s})+\sum_{d=1}^{\infty}\frac{1}{P_{(d+2)}^s}\times((\sum_{c=1}^{d}\frac{1}{(P_{(c)}^2)^s})+\sum_{b=1}^{d}\frac{1}{P_{(b+1)}^s}\times\sum_{a=1}^{b}\frac{1}{P_{(a)}^s}))+...\times((\sum_{e=1}^{g}\frac{1}{(P_{(e)}^3)^s})+\sum_{d=1}^{g}\frac{1}{P_{(d+2)}^s}\times((\sum_{c=1}^{d}\frac{1}{(P_{(c)}^2)^s})+\sum_{b=1}^{d}\frac{1}{P_{(b+1)}^s}\times\sum_{a=1}^{b}\frac{1}{P_{(a)}^s})).$$

    Il y a encore une erreur, quel prise de tête, j'y suis presque.

  • Fly7
    Modifié (December 2021)
    Je n'ai pas fini, je pose mon brouillon.
    C'est que ça prend du temps de mettre, de l'ordonner dans le tableau open office.
    Quand j'aurais le temps je peaufinerai.
    Je n'arrive pas à avoir un cadre noir dans mes cellules.

    Produit nul.$$\sum_{k=1}^{\infty}\frac{1}{k^s}>1$$

    Un nombre premier.$$\sum_{k=1}^{\infty}\frac{1}{k^s}>\sum_{b=1}^{\infty}\sum_{a=1}^{\infty}\frac{1}{(P_{(a)}^b)^s}$$

    Deux$$\sum_{k=1}^{\infty}\frac{1}{k^s}>\sum_{d=1}^{\infty}\sum_{c=1}^{d}\sum_{b=1}^{\infty}\frac{1}{(P_{(b+1)}^c)^s}\times\sum_{a=1}^{b}\frac{1}{(P_{(a)}^{d-c+1})^s}$$
    Trois $$\sum_{k=1}^{\infty}\frac{1}{k^s}>\sum_{e=1}^{\infty}\sum_{d=1}^{e}\sum_{c=1}^{\infty}\frac{1}{(P_{(c+2)}^d)^s}\times\sum_{b=1}^{c}\frac{1}{(P_{(b+1)}^{e-d+1})^s}\times\sum_{a=1}^{b}\frac{1}{(P_{(a)}^{e-d+1})^s}$$

    Pour trois je ne suis pas encore sur.

    Ils me faut un support visuel pour voir si c'est bon ou pas.
  • Boécien
    Modifié (December 2021)
    Quelques trivialités supplémentaires en m'inspirant de la page donnée par Etanche, on a sauf erreurs :
    \begin{align*}\sum_{k\geq2}\left(\zeta(k)-1-\frac{1}{2^{k}}-\frac{1}{3^{k}}-\cdots-\frac{1}{n^{k}}\right)&=\frac{1}{n} \\
    \sum_{k\geq2}\frac{1}{k}\left(\zeta(k)-1-\frac{1}{2^{k}}-\frac{1}{3^{k}}-\cdots-\frac{1}{n^{k}}\right)&=H(n)-\log(n)-\gamma\end{align*}
  • Fly7
    Modifié (December 2021)
    Produit nul.$$\sum_{k=1}^{\infty}\frac{1}{k^s}>1$$
    Un nombre premier.$$\sum_{k=1}^{\infty}\frac{1}{k^s}>\sum_{b=1}^{\infty}\sum_{a=1}^{\infty}\frac{1}{(P_{(a)}^b)^s}$$
    Deux$$\sum_{k=1}^{\infty}\frac{1}{k^s}>\sum_{d=1}^{\infty}\sum_{c=1}^{d}\sum_{b=1}^{\infty}\frac{1}{(P_{(b+1)}^{d-c+1})^s}\times\sum_{a=1}^{b}\frac{1}{(P_{(a)}^{c})^s}$$
    Trois$$\sum_{k=1}^{\infty}\frac{1}{k^s}>\sum_{g=1}^{\infty}\sum_{f=1}^{e}\sum_{e=1}^{\infty}\frac{1}{(P_{(e+2)}^{e-f+1})^s}\times\sum_{d=1}^{f}\sum_{c=1}^{d}\sum_{b=1}^{e}\frac{1}{(P_{(b+1)}^{d-c+1})^s}\times\sum_{a=1}^{b}\frac{1}{(P_{(a)}^{c})^s}$$

    Je ne suis pas encore sur de mon coups pour trois mais ci c'est bon, il ne devrait plus y avoir d’obstacle pour quatre jusqu’à l'infini.

  • Fly7
    Modifié (December 2021)
    N'ayant pas eu d’obstruction et étant incapable de vérifier, je postule.$$\sum_{k=1}^{\infty}\frac{1}{k^s}=1+\sum_{b=1}^{\infty}\sum_{a=1}^{\infty}\frac{1}{(P_{(a)}^b)^s}+\sum_{d=1}^{\infty}\sum_{c=1}^{d}\sum_{b=1}^{\infty}\frac{1}{(P_{(b+1)}^{d-c+1})^s}\times\sum_{a=1}^{b}\frac{1}{(P_{(a)}^{c})^s}+\sum_{g=1}^{\infty}\sum_{f=1}^{e}\sum_{e=1}^{\infty}\frac{1}{(P_{(e+2)}^{e-f+1})^s}\times\sum_{d=1}^{f}\sum_{c=1}^{d}\sum_{b=1}^{e}\frac{1}{(P_{(b+1)}^{d-c+1})^s}\times\sum_{a=1}^{b}\frac{1}{(P_{(a)}^{c})^s}+...\times\sum_{j=1}^{\infty}\sum_{i=1}^{h}\sum_{h=1}^{\infty}\frac{1}{(P_{(h+3)}^{h-i+1})^s}\sum_{g=1}^{i}\sum_{f=1}^{e}\sum_{e=1}^{h}\frac{1}{(P_{(e+2)}^{e-f+1})^s}\times\sum_{d=1}^{f}\sum_{c=1}^{d}\sum_{b=1}^{e}\frac{1}{(P_{(b+1)}^{d-c+1})^s}\times\sum_{a=1}^{b}\frac{1}{(P_{(a)}^{c})^s}.$$
  • Fly7
    Modifié (December 2021)

    $$\sum_{k=1}^{\infty}\frac{1}{k^s}=1+\sum_{e=1}^{\infty}\prod_{d=1}^{e}\sum_{c_{[d]}=1}^{c_{[d+1]}}\sum_{b_{[d]}=1}^{c_{[d]}}\sum_{a_{[d]}=1}^{a_{[d+1]}}\frac{1}{(P_{(a_{[d]}+d-1)}^{c_{[d]}-b_{[d]}+1})^s}$$

    Je tente une écriture plus condensé sans être certain du bon formalisme.

    PS: $b_{[1]}$ est une variable, $b_{[2]}$ en est une autre et ainsi de suite.

    PS2:  Je n'ai pas trouvé comment faire en sorte que les variables tendent vers l’infini a la fin de chaque itération du produit $\prod$ pour la borne supérieur a et c.

  • Fly7
    Modifié (December 2021)
    Boécien a dit :
    Quelques trivialités supplémentaires en m'inspirant de la page donnée par Etanche, on a sauf erreurs :
    \begin{align*}\sum_{k\geq2}\left(\zeta(k)-1-\frac{1}{2^{k}}-\frac{1}{3^{k}}-\cdots-\frac{1}{n^{k}}\right)&=\frac{1}{n} \\
    \sum_{k\geq2}\frac{1}{k}\left(\zeta(k)-1-\frac{1}{2^{k}}-\frac{1}{3^{k}}-\cdots-\frac{1}{n^{k}}\right)&=H(n)-\log(n)-\gamma\end{align*}
    Bizarre moi j'airais dit n-1
    et a l'autre $\zeta_{(1)}-1$ oups $H(n)-1$
    Hmm je pense m'etre trompé.
    Petite question $\sum_{k\geq2}=\sum_{k=2}^{\infty}$?
    Bon je corrige $\frac{1}{n}$
    la seconde aucune idée.
  • Fly7
    Modifié (December 2021)
    $$\sum_{k=1}^{\infty}\frac{1}{k^s}=\\\sum_{e=0}^{\infty}\lim_{b_{[1]}\land a_{[1]} \to +\infty}\prod_{d=0}^{e}\sum_{b_{[d+1]}=1}^{b_{[d]}}\sum_{a_{[d+1]}=1}^{a_{[d]}}\frac{1}{(P_{(a_{[d+1]}+e-d)}^{b_{[d]}-b_{[d+1]}+1})^s}=\\\sum_{e=0}^{\infty}\lim_{a_{[\frac{e(e+1)}{2}]} \to +\infty}\prod_{d=0}^{e}(\sum_{b=0}^{e-d}\sum_{a_{[\frac{(e-d)(e-d+1)}{2}-b]}=1}^{a_{[\frac{(e-d)(e-d+1)}{2}-b+1]}})\frac{1}{(P_{(a_{[\frac{(e-d)(e-d+1)}{2}]}+e-d-1)})^s}=\\\sum_{e=0}^{\infty}\prod_{d=0}^{e}\sum_{a_{[d]}=1}^{\infty}\frac{1}{(P_{(\sum_{b=1}^da_{[b]})})^s-1}\\equation 4$$

    Pour $e=0$.

    $$1=\\1=\\1$$

    Pour $e=1$.

    $$\lim_{b_{[1]}\land a_{[1]} \to +\infty}1\times\sum_{b_{[2]}=1}^{b_{[1]}}\sum_{a_{[2]}=1}^{a_{[1]}}\frac{1}{(P_{(a_{[2]})}^{b_{[1]}-b_{[2]}+1})^s}=\\\lim_{a_{[2]} \to +\infty}\times1\sum_{a_{[1]}=1}^{a_{[2]}}\frac{1}{(P_{(a_{[1]})})^s-1}=\\1\times\sum_{a_{[1]}=1}^{\infty}\frac{1}{(P_{(a_{[1]})})^s-1}=\\\frac{1\times(\sum_{a=1}^{\infty}\frac{1}{{(p_{(a)})}^2-1})^1}{1}$$


    Pour $e=2$.

    $$\lim_{b_{[1]}\land a_{[1]} \to +\infty}1\times\sum_{b_{[2]}=1}^{b_{[1]}}\sum_{a_{[2]}=1}^{a_{[1]}}\frac{1}{(P_{(a_{[2]}+1)}^{b_{[1]}-b_{[2]}+1})^s}\times\sum_{b_{[3]}=1}^{b_{[2]}}\sum_{a_{[3]}=1}^{a_{[2]}}\frac{1}{(P_{(a_{[3]})}^{b_{[2]}-b_{[3]}+1})^s}=\\\lim_{a_{[4]} \to +\infty}1\times\sum_{a_{[3]}=1}^{a_{[4]}}\sum_{a_{[2]}=1}^{a_{[3]}}\frac{1}{(P_{(a_{[3]}+1)})^s-1}\times\sum_{a_{[1]}=1}^{a_{[2]}}\frac{1}{(P_{(a_{[1]})})^s-1}=\\1\times\sum_{a_{[1]}=1}^{\infty}\frac{1}{(P_{(a_{[1]})})^s-1}\times\sum_{a_{[2]}=1}^{\infty}\frac{1}{(P_{(a_{[1]}+a_{[2]})})^s-1}=\\\frac{1\times(\sum_{a=1}^{\infty}\frac{1}{{(p_{(a)})}^s-1})^2-\sum_{b=1}^{\infty}\frac{1}{{((p_{(b)})^s-1)}^2}}{2}$$

    Pour $e=3$.

    $$\lim_{b_{[1]}\land a_{[1]} \to +\infty}\times1\sum_{b_{[2]}=1}^{b_{[1]}}\sum_{a_{[2]}=1}^{a_{[1]}}\frac{1}{(P_{(a_{[2]}+2)}^{b_{[1]}-b_{[2]}+1})^s}\times\sum_{b_{[3]}=1}^{b_{[2]}}\sum_{a_{[3]}=1}^{a_{[2]}}\frac{1}{(P_{(a_{[3]}+1)}^{b_{[2]}-b_{[3]}+1})^s}\times\sum_{b_{[4]}=1}^{b_{[3]}}\sum_{a_{[4]}=1}^{a_{[3]}}\frac{1}{(P_{(a_{[4]})}^{b_{[3]}-b_{[4]}+1})^s}=\\\lim_{a_{[7]} \to +\infty}1\times\sum_{a_{[6]}=1}^{a_{[7]}}\sum_{a_{[5]}=1}^{a_{[6]}}\sum_{a_{[4]}=1}^{a_{[5]}}\frac{1}{(P_{(a_{[6]}+2)})^s-1}\times\sum_{a_{[3]}=1}^{a_{[4]}}\sum_{a_{[2]}=1}^{a_{[3]}}\frac{1}{(P_{(a_{[3]}+1)})^s-1}\times\sum_{a_{[1]}=1}^{a_{[2]}}\frac{1}{(P_{(a_{[1]})})^s-1}=\\1\times\sum_{a_{[1]}=1}^{\infty}\frac{1}{(P_{(a_{[1]})})^s-1}\times\sum_{a_{[2]}=1}^{\infty}\frac{1}{(P_{(a_{[1]}+a_{[2]})})^s-1}\times\sum_{a_{[3]}=1}^{\infty}\frac{1}{(P_{(a_{[1]}+a_{[2]}+a_{[3]})})^s-1}=\\\frac{1\times(\sum_{a=1}^{\infty}\frac{1}{{(p_{(a)})}^s-1})^3-\sum_{b=1}^{\infty}\frac{1}{{((p_{(b)})^s-1)}^3}}{Peut-etre trois}$$


Connectez-vous ou Inscrivez-vous pour répondre.